16

𝐖𝐙 𝐊𝐞𝐭𝐡𝐚𝐝 芚曞

604
1
$$\newcommand{BA}[0]{\begin{align*}} \newcommand{BE}[0]{\begin{equation}} \newcommand{bl}[0]{\boldsymbol} \newcommand{D}[0]{\displaystyle} \newcommand{EA}[0]{\end{align*}} \newcommand{EE}[0]{\end{equation}} \newcommand{h}[0]{\boldsymbol{h}} \newcommand{k}[0]{\boldsymbol{k}} \newcommand{L}[0]{\left} \newcommand{l}[0]{\boldsymbol{l}} \newcommand{m}[0]{\boldsymbol{m}} \newcommand{n}[0]{\boldsymbol{n}} \newcommand{R}[0]{\right} $$

$\Large 𝙞𝚗𝚝𝚛𝚘𝚍𝚞𝚌𝚝𝚒𝚘𝚗$
 ${\rm Almkvist}$, ${\rm Borwein}$, ${\rm Bradley}$, ${\rm Granville}$, ${\rm Koecher}$, ${\rm Leshchiner}$,${\rm Rivoal}$ずいったひずたちはれヌタ倀に収束するような収束が速い玚数を䞎えおいたす。
䟋えば${\rm Koecher's~ formula}$

\begin{align*} \sum_{n=0}^\infty x^{2n}\zeta(2n+3) =\sum_{n=1}^\infty \frac{(-1)^{n-1}}{n^3\binom{2n}{n}}\L(\frac{1}{2}+\frac{2}{1-\frac{x^2}{n^2}}\R)\prod_{k=1}^{n-1}\L(1-\frac{x^2}{k^2}\R) \end{align*}

や$\textrm{Bailey-Borwein-Bladley}~\rm formula$

\begin{align*} \sum_{n=0}^\infty x^{2n}\zeta(2n+2)=3\sum_{n=1}^\infty \frac{1}{\binom{2n}{n}(n^2-x^2)}\prod_{k=1}^{n-1}\frac{k^2-4x^2}{k^2-x^2} \end{align*}

ずいうものがありたす。
${\rm Khodabakhsh}$ず${\rm Tatiana~ Hessami~ Pilehrood}$は${\rm WZ~ method }$を甚いおこのような等匏を蚌明したした。これらのれヌタ倀公匏の${\rm WZ~ method }$による蚌明はどのようにしお超幟䜕玚数ず関係しおいるのでしょうか。たたこの関連性によっおどのように䞀般化するこずができるでしょうか。

$\small 𝐓𝐡𝐞~𝐖𝐙~𝐊𝐞𝐭𝐡𝐚𝐝 $

 次のような方向性を持぀各栌子蟺に重みが付いた栌子グラフを考えたす。

どの二点間においおも経路の重みが䞀臎するように蚭定したす(経路䞍倉性)。たたこのような性質を持぀栌子グラフを$\rm PiGG$ず呌ぶこずにしたす。
$\rm PiGG$であるこずを確かめるには各栌子に぀いお考えれば十分です。

隣り合わない頂点どうしを新たに結ぶこずで同じ点集合に別の栌子を䞎えるこずができたす。そしおその栌子グラフは経路䞍倉性を持ちたす。
いた各頂点が$\mathbb Z×\mathbb Z$䞊にある$\rm PiGG$があるずしたす。たた$(i,j)$から$(i+1,j)$たでの重みを$F(i,j)$$(i,j)$から$(i,j+1)$たでの重みを$G(i,j)$ずしたす。

経路䞍倉性を匏で衚せば

\begin{align*} F(i,j)+G(i+1,j)=G(i,j)+F(i,j+1) \end{align*}

ずなりたす。この等匏を満たす$F,G$の組を${\textrm{WZ-pair}}$ず呌びたす。
冒頭の疑問に察しお$F,G$が特定の圢

\begin{align*} \frac{\Gamma(a_1 i+b_1 j+v_1)\cdots\Gamma(a_r i+b_r j+v_r)}{\Gamma(c_1 i+d_1 j+w_1)\cdots\Gamma(c_s i+d_s j+w_s)}X^iY^j \end{align*}

を持っおくれるずうれしいです。ここで$a_n,b_n,c_n,d_n$は敎数$v_n,w_n,X,Y$は耇玠数ずしおいたす。
たたこの圢で衚すこずができるずいうこずは$\frac{F(i+1,j)}{F(i,j)},\frac{F(i,j+1)}{F(i,j)},\frac{F(i,j)}{G(i,j)}$が有理関数になるこずを瀺しおいたす。

この栌子グラフですが栌子の頂点の座暙は敎数でなくおも良いこずに泚意しおください。あくたで栌子蟺の長さが敎数であれば栌子グラフが圢成されるずいうのが本質です。

$\small {\rm Binomial~theorem}$

 $F,G$を次のように定矩したす。

\begin{align*} &F(i,j)=\frac{\Gamma(i+j+2)}{\Gamma(i+2)\Gamma(j+1)}x^i(1-x)^j=\binom{i+j+1}{j}x^i(1-x)^j \\ &G(i,j)=-\frac{\Gamma(i+j+2)}{\Gamma(i+1)\Gamma(j+2)}x^i(1-x)^j=-\binom{i+j+1}{i}x^i(1-x)^j \end{align*}

$F(i,j)+G(i+1,j)=G(i,j)+F(i,j+1)$が成り立぀ので$(F,G)$は${\textrm{WZ-pair}}$です。
次の栌子を考えたす。

経路$P$で和をずるず

\begin{align*} \sum_{i=0}^{m-1}F(i,0)+\sum_{j=0}^{n-1}G(m,j)=\frac{1-x^m}{1-x}-\sum_{j=0}^{n-1}\binom{j+m+1}{m}x^m(1-x)^j \end{align*}

経路$Q$で和をずるず

\begin{align*} \sum_{j=0}^{n-1}G(0,j)+\sum_{i=0}^{m-1}F(i,n)=-\frac{1-(1-x)^n}{x}+\sum_{i=0}^{m-1}\binom{i+n+1}{n}x^i(1-x)^n \end{align*}

ずなりたす。
よっお

\begin{align*} \frac{1-x^m}{1-x}-\sum_{j=0}^{n-1}\binom{j+m+1}{m}x^m(1-x)^j=-\frac{1-(1-x)^n}{x}+\sum_{i=0}^{m-1}\binom{i+n+1}{n}x^i(1-x)^n \end{align*}

ずいう等匏を埗たす。いた$m\to\infty$ずすれば

\begin{align*} \frac{1-0}{1-x}-0=-\frac{1-(1-x)^n}{x}+\sum_{i=0}^{\infty}\binom{i+n+1}{n}x^i(1-x)^n \end{align*}

すなわち

\begin{align*} \sum_{i=0}^{\infty}\binom{i+n+1}{n}x^i(1-x)^n=\frac{1}{1-x}+\frac{1-(1-x)^n}{x} \end{align*}

ずなりたす。

$\small {\rm Shadowing}$

 前述の${\textrm{WZ-pair}}$を修食するにはどうすればよいでしょうか。そこで次の定理を甚いたす。

${\rm Lemma.}$
 $(F(i,j),G(i,j))$が${\textrm{WZ-pair}}$ならば$F,G$が収束するような任意の耇玠数$\alpha,\beta$に察しお$(F(i+\alpha,j+\beta),G(i+\alpha,j+\beta))$は${\textrm{WZ-pair}}$である

これにより

\begin{align*} &F(i,j)=\frac{\Gamma(i+j+\alpha)}{\Gamma(i+1+\alpha)\Gamma(j)}x^i(1-x)^j \\ &G(i,j)=-\frac{\Gamma(i+j+\alpha)}{\Gamma(i+\alpha)\Gamma(j+1)}x^i(1-x)^j \end{align*}

が${\textrm{WZ-pair}}$ずわかりたす。いた$(0,0)$から$(m,0)$たでの重み

\begin{align*} \sum_{i=0}^{m-1}F(i,0) \end{align*}

を考えるずき

\begin{align*} F(i,0)=\frac{\Gamma(i+\alpha)}{\Gamma(i+1+\alpha)\Gamma(0)}x^i \end{align*}

ずなり分母に$\Gamma(0)$が出珟したす。そこで$F,G$に呚期$1$の

\begin{align*} e(j)=(-1)^j\Gamma(j)\Gamma(1-j)=\frac{\pi(-1)^j}{\sin \pi j} \end{align*}

を掛ければ$(e(j)F(i,j),e(j)G(i,j))$は${\textrm{WZ-pair}}$ずなりたす。これを${\rm Shadowing}$ずいいたす。
すなわち

\begin{align*} &F(i,j)=\frac{\Gamma(i+j+\alpha)\Gamma(1-j)}{\Gamma(i+1+\alpha)}x^i(x-1)^j \\ &G(i,j)=\frac{\Gamma(i+j+\alpha)\Gamma(-j)}{\Gamma(i+\alpha)}x^i(x-1)^j \end{align*}

ず改めたす。

${\rm Three~Paths~Way~1}$

 次の経路を考えたす。

赀の経路ず緑の経路の重みを足し合わせるず青の経路の重みず等しくなりたす。たた青経路ず緑経路はそれぞれ暪軞ず瞊軞に沿っお進みたす。
蚌明は省きたすがこの経路で$n\to\infty$ずすれば緑の重みは$0$になりたす。
いた赀経路䞊で$(0,0)$に最も近い栌子点を$(p,-q)$ずし$(kp,-kq)\to ((k+1)p,-(k+1)q)$の重みを$H_{p,q}(k)$ずしたす。
このずき

\begin{align*} \sum_{i=0}^\infty F(i,0)=\sum_{k=0}^\infty H_{p,q}(k) \end{align*}

であり$H_{p,q}(k)$は

\begin{align*} H_{p,q}(k) &=\sum_{i=kp}^{(k+1)p-1}F(i,-(k+1)q)-\sum_{j=-(k+1)q}^{-kq-1}G(kp,j)\\ &=\sum_{i=0}^{p-1}F(i+kp,-(k+1)q)-\sum_{j=0}^{q-1}G(kp,j-(k+1)q) \end{align*}

ずなりたす。$\D F(i,0)=\frac{x^i}{i+\alpha}~$です。$H_{p,q}(k)$はどうなるでしょうか。
䞀般の$H_{p,q}(k)$を求めるのは困難かもしれたせんが具䜓的な$p,q$に察しお蚈算しおみたす。

\begin{align*} \qquad H_{1,1}(k) &=\sum_{i=0}^{0}F(i+k,-(k+1))-\sum_{j=0}^{0}G(k,j-(k+1))\\ &=F(k,-(k+1))-G(k,-(k+1))\\ &=\frac{\Gamma(-1+\alpha)\Gamma(k+2)}{\Gamma(k+\alpha+1)}x^k(x-1)^{-k-1}-\frac{\Gamma(-1+\alpha)\Gamma(k+1)}{\Gamma(k+\alpha)}x^k(x-1)^{-k-1}\\ &=\frac{\Gamma(-1+\alpha)k!}{\Gamma(k+\alpha+1)}x^k(x-1)^{-k-1}(k+1-(k+\alpha))\\ &=-\frac{\Gamma(\alpha)k!}{\Gamma(k+\alpha+1)}x^k(x-1)^{-k-1}\\ &=\frac{1}{\alpha(1-x)}\frac{k!}{(1+\alpha)_k}\L(\frac{x}{x-1}\R)^k \end{align*}

\begin{align*} \qquad H_{2,1}(k) &=\sum_{i=0}^{1}F(i+2k,-(k+1))-\sum_{j=0}^{0}G(2k,j-(k+1))\\ &=F(2k,-(k+1))+F(2k+1,-(k+1))-G(2k,-(k+1))\\ &=\frac{\Gamma(k+\alpha-1)\Gamma(k+2)}{\Gamma(2k+\alpha+1)}x^{2k}(x-1)^{-k-1}+\frac{\Gamma(k+\alpha)\Gamma(k+2)}{\Gamma(2k+\alpha+2)}x^{2k+1}(x-1)^{-k-1}-\frac{\Gamma(k+\alpha-1)\Gamma(k+1)}{\Gamma(2k+\alpha)}x^{2k}(x-1)^{-k-1}\\ &=\frac{\Gamma(k+\alpha-1)k!}{\Gamma(2k+\alpha+2)}x^{2k}(x-1)^{-k-1}((k+1)(2k+\alpha+1)+(k+1)(k+\alpha-1)x-(2k+\alpha)(2k+\alpha+1))\\ &=\frac{\Gamma(k+\alpha-1)k!}{\Gamma(2k+\alpha+2)}x^{2k}(x-1)^{-k-1}((k+1-(2k+\alpha))(2k+\alpha+1)+(k+1)(k+\alpha-1)x)\\ &=\frac{\Gamma(k+\alpha-1)k!}{\Gamma(2k+\alpha+2)}x^{2k}(x-1)^{-k-1}((-k-\alpha+1)(2k+\alpha+1)+(k+1)(k+\alpha-1)x)\\ &=-\frac{\Gamma(k+\alpha)k!}{\Gamma(2k+\alpha+2)}x^{2k}(x-1)^{-k-1}((2k+\alpha+1)-(k+1)x)\\ &=\frac{1}{\alpha(1+\alpha)(1-x)}((2-x)k+1-x+\alpha)\frac{(\alpha)_k k!}{(2+\alpha)_{2k}}\L(\frac{x^2}{x-1}\R)^k \end{align*}
\begin{align*} \qquad H_{1,2}(k) &=\sum_{i=0}^{0}F(i+k,-2(k+1))-\sum_{j=0}^{1}G(k,j-2(k+1))\\ &=F(k,-(2k+2))-G(k,-(2k+2))-G(k,-(2k+1))\\ &=\frac{\Gamma(-k+\alpha-2)\Gamma(2k+3)}{\Gamma(k+\alpha+1)} x^k(x-1)^{-2k-2} -\frac{\Gamma(-k+\alpha-2)\Gamma(2k+2)}{\Gamma(k+\alpha)} x^k(x-1)^{-2k-2} -\frac{\Gamma(-k+\alpha-1)\Gamma(2k+1)}{\Gamma(k+\alpha)} x^k(x-1)^{-2k-1}\\ &=\frac{\Gamma(-k+\alpha-2)(2k)!}{\Gamma(k+\alpha+1)}x^k(x-1)^{-2k-2}((2k+1)(2k+2)-(2k+1)(k+\alpha)-(-k+\alpha-2)(k+\alpha)(x-1))\\ &=\frac{\Gamma(-k+\alpha-2)(2k)!}{\Gamma(k+\alpha+1)}x^k(x-1)^{-2k-2}((2k+1)(k-\alpha+2)-(-k+\alpha-2)(k+\alpha)(x-1))\\ &=-\frac{\Gamma(-k+\alpha-1)(2k)!}{\Gamma(k+\alpha+1)}x^k(x-1)^{-2k-2}(2k+1+(k+\alpha)(x-1))\\ &=-\frac{\Gamma(\alpha-1)}{\Gamma(1+\alpha)}((1+x)k+1+\alpha(x-1))\frac{(\alpha-1)_{-k}(2k)!}{(1+\alpha)_k}\L(\frac{x}{(1-x)^2}\R)^k\frac{1}{(1-x)^2}\\ &=\frac{1}{\alpha(1-\alpha)(1-x)^2}((1+x)k+1+\alpha(x-1))\frac{(2k)!}{(1+\alpha)_k(2-\alpha)_k}\L(\frac{-x}{(1-x)^2}\R)^k\\ \end{align*}
\begin{align*} \qquad H_{2,2}(k) &=\sum_{i=0}^{1}F(i+2k,-2(k+1))-\sum_{j=0}^{1}G(2k,j-2(k+1))\\ &=F(2k,-(2k+2))+F(2k+1,-(2k+2))-G(2k,-(2k+2))-G(2k,-(2k+1))\\ &=\frac{\Gamma(\alpha-2)\Gamma(2k+3)}{\Gamma(2k+\alpha+1)} x^{2k}(x-1)^{-2k-2} +\frac{\Gamma(\alpha-1)\Gamma(2k+3)}{\Gamma(2k+\alpha+2)} x^{2k+1}(x-1)^{-2k-2} -\frac{\Gamma(\alpha-2)\Gamma(2k+2)}{\Gamma(2k+\alpha)} x^{2k}(x-1)^{-2k-2} -\frac{\Gamma(\alpha-1)\Gamma(2k+1)}{\Gamma(2k+\alpha)} x^{2k}(x-1)^{-2k-1}\\ &=\frac{\Gamma(\alpha-2)(2k)!}{\Gamma(2k+\alpha+2)}x^{2k}(x-1)^{-2k-2} ((2k+1)(2k+2)(2k+\alpha+1)+(\alpha-2)(2k+1)(2k+2)x-(2k+1)(2k+\alpha)(2k+\alpha+1)-(\alpha-2)(2k+\alpha)(2k+\alpha+1)(x-1))\\ &=\frac{\Gamma(\alpha-2)(2k)!}{\Gamma(2k+\alpha+2)}x^{2k}(x-1)^{-2k-2} ((2k+1)(2k+\alpha+1)(2-\alpha)+(\alpha-2)(2k+\alpha)(2k+\alpha+1)+(\alpha-2)((2k+1)(2k+2)-(2k+\alpha)(2k+\alpha+1))x)\\ &=\frac{\Gamma(\alpha-2)(2k)!}{\Gamma(2k+\alpha+2)}x^{2k}(x-1)^{-2k-2} ((\alpha-2)(\alpha-1)(2k+\alpha+1)+(\alpha-2)((2k+1)(2k+2)-(2k+\alpha)(2k+\alpha+1))x)\\ &=\frac{\Gamma(\alpha-1)(2k)!}{\Gamma(2k+\alpha+2)}x^{2k}(x-1)^{-2k-2} ((\alpha-1)(2k+\alpha+1)+(1-\alpha)(4k+\alpha+2)x)\\ &=\frac{\Gamma(\alpha)(2k)!}{\Gamma(2k+\alpha+2)}x^{2k}(x-1)^{-2k-2} (2k+\alpha+1-(4k+\alpha+2)x)\\ &=\frac{\Gamma(\alpha)}{\Gamma(2+\alpha)}(2k+\alpha+1-(4k+\alpha+2)x)\frac{(2k)!}{(2+\alpha)_{2k}}\L(\frac{x}{x-1}\R)^{2k}\frac{1}{(1-x)^2}\\ &=\frac{1}{\alpha(1+\alpha)(1-x)^2}(2(1-2x)k+(1-x)(2+\alpha)-1)\frac{(2k)!}{(2+\alpha)_{2k}}\L(\frac{x}{1-x}\R)^{2k} \end{align*}

このようになりたす。たずめるず

<p style=" margin: 0; padding: 0; "
\begin{align*} \sum_{n=0}^\infty \frac{x^n}{n+\alpha} &=\frac{1}{\alpha(1-x)}\sum_{n=0}^\infty \frac{n!}{(1+\alpha)_n}\L(\frac{x}{x-1}\R)^n\\ &=\frac{1}{\alpha(1+\alpha)(1-x)}\sum_{n=0}^\infty ((2-x)n+1-x+\alpha)\frac{(\alpha)_n n!}{(2+\alpha)_{2n}}\L(\frac{x^2}{x-1}\R)^n\\ &=\frac{1}{\alpha(1-\alpha)(1-x)^2}\sum_{n=0}^\infty ((1+x)n+1+\alpha(x-1))\frac{(2n)!}{(1+\alpha)_n(2-\alpha)_n}\L(\frac{-x}{(1-x)^2}\R)^n\\ &=\frac{1}{\alpha(1+\alpha)(1-x)^2}\sum_{n=0}^\infty (2(1-2x)n+(1-x)(2+\alpha)-1)\frac{(2n)!}{(2+\alpha)_{2n}}\L(\frac{x}{1-x}\R)^{2n} \end{align*}


ずなりたす。
䟋えば行目の匏においお$\alpha$で埮分し$x=-1,\alpha=1$を代入するず

\begin{align*} \sum_{n=1}^\infty \frac{(-1)^{n-1}}{n^2} =\sum_{n=1}^\infty \frac{(-1)^{n-1}}{2^nn\binom{2n}{n}}\L(\frac{2}{n}+3(H_{2n}-H_n)\R) \end{align*}

ずいう匏が埗られたす。

${\rm Three~Paths~Way~2}$

 ${\textrm{WZ-pair}}$

\begin{align*} &F(i,j)=\frac{H(i,j)}{i}\\ &G(i,j)=-\frac{H(i,j)}{j}\\ &H(i,j)=\frac{\Gamma(i+j)}{\Gamma(i)\Gamma(j)}z^i(1-z)^j  \L(0< z<\frac{1}{2}\R) \end{align*}

に察しお次の経路を考えたす。

前述ず同様に$n\to\infty$においお緑の重みは$0$に収束したす。
赀の経路䞊で$(s,t)$に最も近い栌子点を$(s+p,t+q)$ずし$(s+kp,t+kq)\to (s+(k+1)p,t+(k+1)q)$の重みを$H_{p,q}(k)$ずしたす。
このずき

\begin{align*} \sum_{m=0}^\infty F(s+m,t)=\sum_{k=0}^\infty H_{p,q}(k) \end{align*}

であり$H_{p,q}(k)$は

\begin{align*} H_{p,q}(k) &=\sum_{n=0}^{p-1}F(s+kp+n,t+kq)+\sum_{n=0}^{q-1}G(s+(k+1)p,t+kq+n)\\ \end{align*}

ずなりたす。実際に$H_{p,q}(k)を蚈算するず$

\begin{align*} H_{1,1}(k)&=\frac{\Gamma(s+t-2)}{\Gamma(s)\Gamma(t)} (A_{1,1}+B_{1,1}(k+1)) \frac{(s+t-2)_{2k+2}}{(s)_{k+1}(t)_{k+1}}z^{s+k}(1-z)^{t+k}\\ H_{2,1}(k)&=\frac{\Gamma(s+t-2)}{\Gamma(s)\Gamma(t)} (A_{2,1}+B_{2,1}(k+1)+C_{2,1}(k+1)^2) \frac{(s+t-2)_{3k+2}}{(s)_{2k+2}(t)_{k+1}}z^{s+2k}(1-z)^{t+k}\\ H_{1,2}(k)&=\frac{\Gamma(s+t-2)}{\Gamma(s)\Gamma(t)} (A_{1,2}+B_{1,2}(k+1)+C_{1,2}(k+1)^2) \frac{(s+t-2)_{3k+2}}{(s)_{k+1}(t)_{2k+2}}z^{s+k}(1-z)^{t+2k} \end{align*}

\begin{align*} \begin{pmatrix}A_{1,1}\\B_{1,1}\end{pmatrix} &=\begin{pmatrix}t-1-(s+t-2)z\\1-2z\end{pmatrix}\\ \begin{pmatrix}A_{2,1}\\B_{2,1}\\C_{2,1} \end{pmatrix} &=\begin{pmatrix}(s-1)(t-1)-(3-s-t)(t-1)z+(s+t-2)(3-s-t)z^2 \\ s+2t-3-(6-s-4t)z+3(5-2s-2t)z^2\\ (1+3z)(2-3z) \end{pmatrix}\\ \begin{pmatrix}A_{1,2}\\B_{1,2}\\C_{1,2}\end{pmatrix} &=\begin{pmatrix}-(t-1)(2-t)+(3-s-t)(s+2t-3)z-(s+t-2)(3-s-t)z^2\\ -2(3-2t)+(24-8s-11t)z-3(5-2s-2t)z^2\\ (1-3z)(2-3z) \end{pmatrix} \end{align*}

ずなりたす。$s,t$に$1$を足し$F,H$の和を$1$からずるように調敎するず

\begin{align*} (s+t)\sum_{m=1}^\infty \frac{(1+s+t)_n}{(1+s)_n}z^{n-1} &=\sum_{k=1}^\infty (A'_{1,1}+B'_{1,1}k)\frac{(s+t)_{2k}}{(1+s)_k(1+t)_k}z^{k-1}(1-z)^{k-1}\\ &=\sum_{k=1}^\infty (A'_{2,1}+B'_{2,1}k+C'_{2,1}k^2)\frac{(s+t)_{3k-1}}{(1+s)_{2k}(1+t)_k}z^{2k-2}(1-z)^{k-1}\\ &=\sum_{k=1}^\infty (A'_{1,2}+B'_{1,2}k+C'_{1,2}k^2)\frac{(s+t)_{3k-1}}{(1+s)_k(1+t)_{2k}}z^{k-1}(1-z)^{2k-2}\\ \end{align*}

\begin{align*} \begin{pmatrix}A'_{1,1}\\B'_{1,1}\end{pmatrix} &=\begin{pmatrix}t-(s+t)z\\1-2z\end{pmatrix}\\ \begin{pmatrix}A'_{2,1}\\B'_{2,1}\\C'_{2,1} \end{pmatrix} &=\begin{pmatrix}st-t(1-s-t)z+(s+t)(1-s-t)z^2 \\ s+2t-(1-s-4t)z+3(1-2s-2t)z^2\\ (1+3z)(2-3z) \end{pmatrix}\\ \begin{pmatrix}A'_{1,2}\\B'_{1,2}\\C'_{1,2}\end{pmatrix} &=\begin{pmatrix}-t(1-t)+(1-s-t)(s+2t)z-(s+t)(1-s-t)z^2\\ -2(1-2t)+(5-8s-11t)z-3(1-2s-2t)z^2\\ (1-3z)(2-3z) \end{pmatrix} \end{align*}

$z^s(1-z)^t$を陀いたので$z$を負に拡匵できたす。すべおの等匏が成り立぀にはより狭い

\begin{align*} -\frac{1}{3}\L(2-(-1+\sqrt{2})^{-\frac{2}{3}}-(-1+\sqrt{2})^{\frac{2}{3}}\R)\le z<\frac{1}{3} \end{align*}

である必芁がありたす。

${\rm telescoping~sum}$

 経路䞍倉性

\begin{align*} F(i,j)-F(i,j+1)=G(i,j)-G(i+1,j) \end{align*}

においお$j$に$j+1,j+2,\cdots,j+m$を代入したものを蟺蟺足し合わせるず

\begin{align*} F(i,j)-F(i,j+m+1)=\sum_{n=0}^m(G(i,j+n)-G(i+1,j+n)) \end{align*}

ずなりたす。いた$F(i,\infty)=0$ず仮定すれば

\begin{align*} F(i,j)=\sum_{n=0}^\infty (G(i,j+n)-G(i+1,j+n)) \end{align*}

ずなりたす。
同様に$i$に$i+1,i+2,\cdots,i+m$を代入しお

\begin{align*} \sum_{n=0}^m(F(i+n,j)-F(i+n,j+1))=G(i,j)-G(i+m+1,j) \end{align*}

ずなり$G(\infty,j)=0$ずすれば

\begin{align*} \sum_{n=0}^\infty (F(i+n,j)-F(i+n,j+1))=G(i,j) \end{align*}

ずなりたす。ここからさらに$j$に$j+1,j+2,\cdots,j+m$を代入しお足し合わせれば

\begin{align*} \sum_{n=0}^\infty (F(i+n,j)-F(i+n,j+m+1))=\sum_{n=0}^mG(i,j+n) \end{align*}

ずなり$\D\lim_{m\to\infty}\sum_{n=0}^\infty F(i+n,j+m)=0$ず仮定すれば

\begin{align*} \sum_{n=0}^\infty F(i+n,j)=\sum_{n=0}^\infty G(i,j+n) \end{align*}

ずなりたす。

$\textrm{Markov-WZ}\rm~method$

 䞀般に$\rm hypergeometric~term$ずしおの${\textrm{WZ-pair}}$を芋぀けるこずは私にずっおは難しいものです。しかしその$\rm term$にパラメヌタの倚項匏を掛けたものを芋぀けるためのアルゎリズムがありそれにより原理ずしおは容易に${\textrm{WZ-pair}}$を芋぀けるこずができたした。
 そのアルゎリズムは

$m,n$の関数$H(m,n)$をテキトヌに決める
$\BA F(m,n)&=(A_0(n)+mA_1(n)+\cdots+m^pA_p(n))H(m,n)\\ G(m,n)&=(B_0(n)+mB_1(n)+\cdots+m^qB_q(n))H(m,n)\EA$ず定矩する
$F(m,n)-F(m,n+1)=G(m,n)-G(m+1,n)$に代入し$(m+1)^r$の係数比范から$A_1(n),\cdots,B_q(n)$を求める

です。無限玚数ずしおの等匏を埗たいずいうずきは$F(\infty,x)$や$G(x,\infty)$などがうたく$0$に収束するように$H(m,n)$を決定する必芁がありたす。
具䜓䟋で蚈算しおみたす。
たず

\begin{align*} H(m,n)=\L(\frac{m!n!}{(m+n)!}\R)^2 \end{align*}

ず定矩し

\begin{align*} F(m,n)&=A(n)H(m,n)\\ G(m,n)&=\big(B(n)+mC(n)\big)H(m,n) \end{align*}

ず定矩したす。
このずき

\begin{align*} F(m,n)-F(m,n+1) &=A(n)H(m,n)-A(n+1)H(m,n+1)\\ &=A(n)H(m,n)-A(n+1)\frac{(n+1)^2}{(m+n+1)^2}H(m,n)\\ &=\Big((m+n+1)^2A(n)-(n+1)^2A(n+1)\Big)\frac{H(m,n)}{(m+n+1)^2}\\ &=\Big(n^2A(n)-(n+1)^2A(n+1)+2(m+1)nA(n)+(m+1)^2A(n)\Big)\frac{H(m,n)}{(m+n+1)^2} \end{align*}

たた

\begin{align*}\qquad G(m,n)-G(m+1,n) &=\big(B(n)+mC(n)\big)H(m,n)-\big(B(n)+(m+1)C(n)\big)H(m+1,n)\\ &=\big(B(n)-C(n)+(m+1)C(n)\big)H(m,n)-\big(B(n)+(m+1)C(n)\big)\frac{(m+1)^2}{(m+n+1)^2}H(m,n)\\ &=\Big((m+n+1)^2\big(B(n)-C(n)+(m+1)C(n)\big)-(m+1)^2\big(B(n)+(m+1)C(n)\big)\Big)\frac{H(m,n)}{(m+n+1)^2}\\ &=\Big(\big(n^2+2(m+1)n+(m+1)^2\big)\big(B(n)-C(n)+(m+1)C(n)\big)-(m+1)^2B(n)-(m+1)^3C(n)\Big)\frac{H(m,n)}{(m+n+1)^2}\\ &=\Big(n^2\big(B(n)-C(n)\big)+\big(2n(B(n)-C(n))+n^2C(n)\big)(m+1)+\big(B(n)-C(n)+2nC(n)\big)(m+1)^2+(m+1)^3C(n)-(m+1)^2B(n)-(m+1)^3C(n)\Big)\frac{H(m,n)}{(m+n+1)^2}\\ &=\Big(n^2B(n)-n^2C(n)+\big(2nB(n)+(n^2-2n)C(n)\big)(m+1)+(m+1)^2(2n-1)C(n)\Big)\frac{H(m,n)}{(m+n+1)^2} \end{align*}

ずなりたす。係数比范により

\begin{align*} \begin{cases} &n^2A(n)-(n+1)^2A(n+1)=n^2B(n)-n^2C(n)\\ &2nA(n)=2nB(n)+(n^2-2n)C(n)\\ &A(n)=(2n-1)C(n) \end{cases} \end{align*}

を埗たす。これを解くず

\begin{align*} A(n)=\frac{2(2n-1)}{n^3\binom{2n}{n}}A(1)~, B(n)=\frac{3nA(n)}{2(2n-1)}~, C(n)=\frac{A(n)}{2n-1} \end{align*}

ずなりたす。$A(1)$は$F$ず$G$の比に圱響しないので適圓に$1$ずしおよいです。
すなわち

\begin{align*} F(m,n)&=\frac{2(2n-1)}{n^3\binom{2n}{n}}\L(\frac{m!n!}{(m+n)!}\R)^2\\ G(m,n)&=\frac{2m+3n}{n^3\binom{2n}{n}}\L(\frac{m!n!}{(m+n)!}\R)^2 \end{align*}

が${\textrm{WZ-pair}}$であるこずがわかりたした。
実はこの方法は$\textrm{Markov-WZ}\rm~method$ず呌ばれるらしいです。
この${\textrm{WZ-pair}}$に察しお$\rm telescoping~sum$を考えるず

\begin{align*} \sum_{m=0}^\infty F(m+\alpha,\beta)=\sum_{n=0}^\infty G(\alpha,n+\beta) \end{align*}

すなわち

\begin{align*} \sum_{m=0}^\infty \frac{(1+\alpha)_m^2}{(1+\alpha+\beta)_m^2} =\frac{\beta^4\Gamma(\beta)}{2(2\beta-1)}\sum_{n=0}^\infty\frac{2\alpha+3\beta+3n}{(\beta+n)^3}\frac{(1+\beta)_n^4}{(1+2\beta)_{2n}(1+\alpha+\beta)_n^2}      \L(\beta>\frac{1}{2}\R) \end{align*}

ずなりたす。$\beta=1$の堎合は

\begin{align*} \sum_{n=1}^\infty \frac{1}{(n+\alpha)^2}=\sum_{n=1}^\infty\frac{3n+2\alpha}{n^3\binom{2n}{n}}\frac{n!^2}{(1+\alpha)_n^2} \end{align*}

ずなりたす。
たた$\rm three~paths~way$により

\begin{align*} \sum_{n=0}^\infty F(n+\alpha,\beta)=\sum_{n=0}^\infty H_{p,q}(n) \end{align*}
\begin{align*} H_{p,q}(n)=\sum_{j=0}^{p-1}F(\alpha+pn+j,\beta+qn)+\sum_{k=0}^{q-1}G(\alpha+pn+p,\beta+qn+k) \end{align*}

ずなり実際に$H_{1,1}(n)$を蚈算するず

\begin{align*} H_{1,1}(n)=\frac{\Gamma^2(1+\alpha)\Gamma^4(1+\beta)}{\Gamma(1+2\beta)\Gamma^2(1+\alpha+\beta)}\frac{A+Bn+Cn^2+21n^3}{(\beta+n)^3}\frac{(1+\alpha)_n^2(1+\beta)_n^4}{(1+2\beta)_{2n}(1+\alpha+\beta)_{2n}^2} \end{align*}
\begin{align*} \begin{pmatrix}A\\B\\C\end{pmatrix} =\begin{pmatrix}2(2\beta-1)(1+\alpha+\beta)^2+(2+2\alpha+3\beta)(1+\alpha)^2\\ 8(2\beta-1)(1+\alpha+\beta)+4(1+\alpha+\beta)^2+2(2+2\alpha+3\beta)(1+\alpha)+5(1+\alpha)^2\\ 8(2\beta-1)+16(1+\alpha+\beta)+10(1+\alpha)+2+2\alpha+3\beta \end{pmatrix} \end{align*}

ずなりたした。$H_{1,2}(n),~H_{2,1}(n)$以降は蚈算量が倚すぎお心が折れたした。
$\alpha$ず$\beta$に具䜓的な数倀を代入するず少し蚈算しやすくなるので$\alpha=0,~\beta=1$で蚈算しおみるず

\begin{align*} H_{1,1}(n-1)&=\frac{21n-8}{n^3\binom{2n}{n}^3}\\ H_{1,2}(n-1)&=\frac{145n^2-104n+18}{2n^3(2n-1)\binom{2n}{n}\binom{3n}{n}^2}\\ H_{2,1}(n-1)&=\frac{9}{4}\frac{80n^3-105n^2+44n-6}{n^2(2n-1)^3\binom{3n}{n}^2\binom{4n-2}{2n-1}}+\frac{1}{n^2\binom{3n}{n}^2\binom{4n}{2n}}\\ H_{2,2}(n-1)&=\frac{4(680n^5-1236n^4+846n^3-281n^2+46n-3)}{(2n-1)^5(4n-1)^2\binom{4n-2}{2n-1}\binom{4n}{2n}^2}+\frac{5}{4n^2\binom{4n}{2n}^3} \end{align*}

$p+q$が倧きくなるに぀れお収束が速くなりたす。
なんずいうかコンピュヌタが党郚蚈算しおくれるみたいなこずはできないのでしょうか。流石に手蚈算はキツいです。もしできたら

\begin{align*} H(m,n)=\L(\frac{m!n!}{(m+n)!}\R)^5 \end{align*}

のような堎合の${\textrm{WZ-pair}}$がどのようなかたちになるかもわかりたすが。

${\rm parameterized}$

たず

\begin{align*} H(m,n)=\frac{\Gamma(a+m)\Gamma(b+m)}{\Gamma(c+m+n)\Gamma(d+m+n)} \end{align*}

ず定矩したす。さらに

\begin{align*} &F(m,n)=A(n)H(m,n)\\ &G(m,n)=\big(B(n)+mC(n)\big)H(m,n) \end{align*}

ず定矩したす。

\begin{align*} F(m,n)-F(m,n+1) &=A(n)H(m,n)-A(n+1)H(m,n+1)\\ &=A(n)H(m,n)-A(n+1)\frac{H(m,n)}{(c+m+n)(d+m+n)}\\ &=\big((c+m+n)(d+m+n)A(n)-A(n+1)\big)\frac{H(m,n)}{(c+m+n)(d+m+n)}\\ &=\big(m^2A(n)+(c+d+2n)mA(n)+(c+n)(d+n)A(n)-A(n+1)\big)\frac{H(m,n)}{(c+m+n)(d+m+n)} \end{align*}
\begin{align*}\qquad\qquad G(m,n)-G(m+1,n) &=\big(B(n)+mC(n)\big)H(m,n)-\big(B(n)+(m+1)C(n)\big)\frac{(a+m)(b+m)H(m,n)}{(c+m+n)(d+m+n)}\\ &=\Big((c+m+n)(d+m+n)\big(B(n)+mC(n)\big)-(a+m)(b+m)\big(B(n)+(m+1)C(n)\big)\Big)\frac{H(m,n)}{(c+m+n)(d+m+n)}\\ &=\L(\begin{matrix}\big((c+n)(d+n)+(c+d+2n)m+m^2\big)\big(B(n)+mC(n)\big)\\ -\big(ab+(a+b)m+m^2\big)\big(B(n)+C(n)+mC(n)\big)\end{matrix}\R)\frac{H(m,n)}{(c+m+n)(d+m+n)}\\ &=\Big((c+d-a-b-1+2n)m^2C(n)+\big((c+d-a-b+2n)B(n)+((c+n)(d+n)-a-b-ab)\big)mC(n)+\big((c+n)(d+n)-ab\big)B(n)-abC(n)\Big)\frac{H(m,n)}{(c+m+n)(d+m+n)} \end{align*}

すなわち$(F,G)$が${\textrm{WZ-pair}}$ずなるように

\begin{align*} \begin{cases} A(n)=(c+d-a-b-1+2n)C(n)\\ (c+d+2n)A(n)=(c+d-a-b+2n)B(n)+\big((c+n)(d+n)-a-b-ab\big)C(n)\\ (c+n)(d+n)A(n)-A(n+1)=\big((c+n)(d+n)-ab\big)B(n)-abC(n) \end{cases} \end{align*}

を解くず

\begin{align*} \begin{cases}\D A(n)=\frac{(c-a)_n(c-b)_n(d-a)_n(d-b)_n}{(c+d-a-b-1)_{2n}}\\ \D B(n)=\frac{3n^2+pn+q}{(c+d-a-b-1+2n)(c+d-a-b+2n)}A(n)\qquad\L(\begin{matrix}p=3c+3d-2a-2b-2\\q=(c+d)(c+d-a-b-1)+a+b+ab-cd\end{matrix}\R) \\ \D C(n)=\frac{A(n)}{c+d-a-b-1+2n}\\ \end{cases} \end{align*}

ずなりたす。
ここで

\begin{align*}\qquad\qquad \sum_{m=0}^\infty F(m+\alpha,1+\beta) &=\sum_{m=0}^\infty \frac{(c-a)_{1+\beta}(c-b)_{1+\beta}(d-a)_{1+\beta}(d-b)_{1+\beta}}{(c+d-a-b-1)_{2+2\beta}}\frac{\Gamma(a+\alpha+m)\Gamma(b+\alpha+m)}{\Gamma(1+c+\alpha+m)\Gamma(1+d+\alpha+m)}\\ &=\frac{\Gamma(c+d-a-b-1)\Gamma(c-a+1+\beta)\Gamma(c-b+1+\beta)\Gamma(d-a+1+\beta)\Gamma(d-b+1+\beta)\Gamma(a+\alpha)\Gamma(b+\alpha)}{\Gamma(c-a)\Gamma(c-b)\Gamma(d-a)\Gamma(d-b)\Gamma(c+d-a-b+1+2\beta)\Gamma(c+\alpha)\Gamma(d+\alpha)}\sum_{m=1}^\infty \frac{1}{(m-1+a+\alpha)(m-1+b+\alpha)}\frac{(a+\alpha)_m(b+\alpha)_m}{(c+\alpha)_m(d+\alpha)_m} \end{align*}

たた

\begin{align*}\qquad\qquad \sum_{n=1}^\infty G(\alpha,n+\beta) &=\sum_{n=1}^\infty \L(\frac{3(n+\beta)^2+p(n+\beta)+q}{(c+d-a-b-1+2\beta+2n)(c+d-a-b+2\beta+2n)}+\frac{\alpha}{c+d-a-b-1+2\beta+2n}\R)\frac{(c-a)_{n+\beta}(c-b)_{n+\beta}(d-a)_{n+\beta}(d-b)_{n+\beta}}{(c+d-a-b-1)_{2n+2\beta}}\frac{\Gamma(a+\alpha)\Gamma(b+\alpha)}{\Gamma(c+\alpha+\beta+n)\Gamma(d+\alpha+\beta+n)}\\ &=\frac{\Gamma(c-a+\beta)\Gamma(c-b+\beta)\Gamma(d-a+\beta)\Gamma(d-b+\beta)\Gamma(c+d-a-b-1)\Gamma(a+\alpha)\Gamma(b+\alpha)}{\Gamma(c-a)\Gamma(c-b)\Gamma(d-a)\Gamma(d-b)\Gamma(c+d-a-b+1+2\beta)\Gamma(c+\alpha+\beta)\Gamma(d+\alpha+\beta)} \sum_{n=1}^\infty \big(3n^2+(2\alpha+6\beta+p)n+3\beta^2+p\beta+q+\alpha(c+d-a-b+2\beta)\big) \frac{(c-a+\beta)_n(c-b+\beta)_n(d-a+\beta)_n(d-b+\beta)_n}{(c+d-a-b+1+2\beta)_{2n}(c+\alpha+\beta)_n(d+\alpha+\beta)_n} \end{align*}

ずなるので

 

\begin{align*} &\sum_{m=1}^\infty \frac{1}{(m-1+a+\alpha)(m-1+b+\alpha)}\frac{(a+\alpha)_m(b+\alpha)_m}{(c+\alpha)_m(d+\alpha)_m}\\ =&\frac{\Gamma(c+\alpha)\Gamma(d+\alpha)}{\Gamma(c+\alpha+\beta)\Gamma(d+\alpha+\beta)} \sum_{n=1}^\infty \big(3n^2+(2\alpha+6\beta+p)n+3\beta^2+p\beta+q+\alpha(c+d-a-b+2\beta)\big) \frac{(1+c-a+\beta)_{n-1}(1+c-b+\beta)_{n-1}(1+d-a+\beta)_{n-1}(1+d-b+\beta)_{n-1}}{(c+d-a-b+1+2\beta)_{2n}(c+\alpha+\beta)_n(d+\alpha+\beta)_n} \end{align*}
$(p=3c+3d-2a-2b-2,~q=(c+d)(c+d-a-b-1)+a+b+ab-cd)$

 
$c=a,~d=b,~\alpha=\beta=0$ずするず$p=a+b-2,~q=0 $ずなり

\begin{align*} \sum_{m=1}^\infty \frac{1}{(m-1+a)(m-1+b)} =\sum_{n=1}^\infty \frac{3n+a+b-2}{n\binom{2n}{n}}\frac{(1+a-b)_{n-1}(1-a+b)_{n-1}}{(a)_n(b)_n} \end{align*}

$a\to1-a,~b\to1-b$ずすれば

\begin{align*} \sum_{m=1}^\infty \frac{1}{(m-a)(m-b)} =\sum_{n=1}^\infty \frac{3n-a-b}{n\binom{2n}{n}}\frac{(1+a-b)_{n-1}(1-a+b)_{n-1}}{(1-a)_n(1-b)_n} \end{align*}

さらに$b=-a$ずすれば

\begin{align*} \sum_{m=1}^\infty \frac{1}{m^2-a^2} &=\sum_{n=1}^\infty \frac{3}{\binom{2n}{n}}\frac{(1+2a)_{n-1}(1-2a)_{n-1}}{(1+a)_n(1-a)_n}\\ &=\sum_{n=1}^\infty \frac{3}{\binom{2n}{n}(n^2-a^2)}\prod_{k=1}^{n-1}\frac{k^2-4a^2}{k^2-a^2} \end{align*}

これは冒頭の公匏です。

${\rm accelerated~series~for~}\zeta(3)$

たず$m,n\ge 0$ずしたす。

\begin{align*} H(m,n)=\L(\frac{n!}{(m+n+1)!}\R)^4 \end{align*}

ずし

\begin{align*} &F(m,n)=(A_m+nB_m+n^2C_m+n^3D_m)H(m,n)\\ &G(m,n)=(P_m+(n+1)Q_m+(n+1)^2R_m)H(m,n) \end{align*}

ず定矩したす。

\begin{align*} F(m,n)-F(m,n+1) &=(A_m+nB_m+n^2C_m+n^3D_m)H(m,n)-(A_m+(n+1)B_m+(n+1)^2C_m+(n+1)^3D_m)H(m,n+1)\\ &=((m+n+2)^4(A_m+nB_m+n^2C_m+n^3D_m)-(n+1)^4(A_m+(n+1)B_m+(n+1)^2C_m+(n+1)^3D_m))\frac{H(m,n)}{(m+n+2)^4} \end{align*}
\begin{align*} G(m,n)-G(m+1,n) &=(P_m+(n+1)Q_m+(n+1)^2R_m)H(m,n)-(P_{m+1}+(n+1)Q_{m+1}+(n+1)^2R_{m+1})H(m+1,n)\\ &=((m+n+2)^4(P_m+(n+1)Q_m+(n+1)^2R_m)-(P_{m+1}+(n+1)Q_{m+1}+(n+1)^2R_{m+1}))\frac{H(m,n)}{(m+n+2)^4} \end{align*}

なので

\begin{align*} &(m+n+2)^4(A_m+nB_m+n^2C_m+n^3D_m)-(n+1)^4(A_m+(n+1)B_m+(n+1)^2C_m+(n+1)^3D_m)\\ =&(m+n+2)^4(P_m+(n+1)Q_m+(n+1)^2R_m)-(P_{m+1}+(n+1)Q_{m+1}+(n+1)^2R_{m+1}) \end{align*}

が垞に成り立぀ように$(n+1)^r$の係数を比范するず

\begin{align*} \begin{cases} &[~1~]\qquad (4M-3)D=R\\ &[~2~]\qquad (4M-2)C+(6M^2-12M+3)D=Q+4MR\\ &[~3~]\qquad (4M-1)B+(6M^2-8M+1)C+(4M^3-18M^2+12M-1)D=P+4MQ+6M^2R\\ &[~4~]\qquad 4MA+(6M^2-4M)B+(4M^3-12M^2-4M)C+(M^4-12M^3+18M^2-4M)D=4MP+6M^2Q+4M^3R\\ &[~5~]\qquad 6M^2A+(4M^3-6M^2)B+(M^4-8M^3+6M^2)C-(3M^4-12M^3+6M^2)D=6M^2P+4M^3Q+M^4R-R'\\ &[~6~]\qquad 4M^3A+(M^4-4M^3)B-(2M^4-4M^3)C+(3M^4-4M^3)D=4M^3P+M^4Q-Q'\\ &[~7~]\qquad M^4(A-B+C-D)=M^4P-P' \end{cases} \end{align*}

ただし簡朔の為$X_m:=X,~X_{m+1}:=X',~M:=m+1$ずしおいたす。$F,G$での$n$の次数は倉数の個数ず連立する挞化匏の個数が䞀臎するように調敎しおいたす。
これらを少し䞁寧に解いおいきたす。たず$[~1~]$ず$[~2~]$から$R$を消去するず

\begin{align*} [~2'~]\qquad Q=(4M-2)C-(10M^2-3)D \end{align*}

を埗たす。$[~1~],~[~2'~]$を$[~3~]$に代入するず

\begin{align*} [~3'~]\qquad P=(4M-1)B-(10M^2-1)C+(20M^3-1)D \end{align*}

を埗たす。$[~4~]$の䞡蟺を$M$で割ったものに$[~1~],~[~2'~],~[~3'~]$を代入するず

\begin{align*} [~4'~]\qquad 4A=10MB-20M^2C+35M^3D \end{align*}

を埗たす。$[~1~],~[~2'~],~[~3'~],~[~4'~]$を$[~5~]$に代入するず

\begin{align*} [~5'~]\qquad 2(4M+1)D'=10M^3B-30M^4C+63M^5D \end{align*}

を埗たす。$[~1~],~[~2'~],~[~3'~],~[~4'~]$を$[~6~]$に代入するず

\begin{align*} [~6'~]\qquad 2(4M+2)C'-2(10M^2+20M+7)D'=10M^4B-32M^5C+70M^6D \end{align*}

を埗たす。$M×[~5'~]-[~6'~]$より

\begin{align*} [~8~]\qquad 2C'-7(M+1)D'=-\frac{M^5}{2(2M+1)}(2C-7MD) \end{align*}

これず$[~2'~]$より$Q_0=2C_0-7D_0$ずなるこずから

\begin{align*} [~8'~]\qquad 2C-7MD=\frac{(-1)^mm!^6}{(2m+1)!}Q_0 \end{align*}

を埗たす。$[~8'~]$ず$[~5'~]$から$C$を消去するず

\begin{align*} [~9~]\qquad B=\frac{21}{5}M^2D+\frac{4M+1}{5M^3}D'+\frac{3}{2}\frac{(-1)^mm!^6M}{(2m+1)!}Q_0 \end{align*}

を埗たす。$[~4'~],~[~8'~],~[~9~]$から$A$を$D$で衚すず

\begin{align*} [10]\qquad A=\frac{7}{4}M^3D+\frac{4M+1}{2M^2}D'+\frac{5}{4}\frac{(-1)^mm!^6M^2}{(2m+1)!}Q_0 \end{align*}

を埗たす。これで$A,B,C,P,Q$を$D$で衚すこずができたした。
すなわち

\begin{align*} &A=\frac{7}{4}M^3D+\frac{4M+1}{2M^2}D'+\frac{5}{4}M^2V\\ &B=\frac{21}{5}M^2D+\frac{4M+1}{5M^3}D'+\frac{3}{2}MV\\ &C=\frac{7}{2}MD+\frac{1}{2}V\\ &P=-\frac{1}{10}(182M^3-158M^2-35M+10)D+\frac{16M^2-1}{5M^3}D'+\frac{1}{2}(2M-1)(M-1)V\\ &Q=(4M-3)(M-1)D+(2M-1)V\\ &R=(4M-3)D \end{align*}

ずなりたす。ただし簡朔の為$V=\dfrac{(-1)^mm!^6}{(2m+1)!}Q_0$ずしおいたす。
$D$の初期条件をみたす。䞊匏から

\begin{align*} &D_0=R_0\\ &D_1=\frac{1}{3}P_0-\frac{1}{30}D_0 \end{align*}

がわかりたす。たた$D_0$ず$D_1$が決たれば$A$$R$がすべお決たるこずがわかりたす。これらの$A$$R$を$D$で衚したものを$[~7~]$に代入するこずで䞉項間挞化匏が珟れたす。この挞化匏の䞀般項の明瀺匏を求めるのは非垞に困難であるず予想したす。
ずなれば$D$が垞に$0$ずなればうれしいです。そのために$R_0=P_0=0$ずしたす。たた$Q_0=1$ずしたす。
これにより$D$は垞に$0$ずなり

\begin{align*} &A=\frac{5}{4}M^2V\\ &B=\frac{3}{2}MV\\ &C=\frac{1}{2}V\\ &D=0\\ &P=\frac{1}{2}(2M-1)(M-1)V\\ &Q=(2M-1)V\\ &R=0 \end{align*}

ずなりたす。これらを$F,G$に代入するず

\begin{align*} &F(m,n)=\frac{5(m+1)^2+6(m+1)n+2n^2}{4}\frac{(-1)^mm!^6}{(2m+1)!}\L(\frac{n!}{(m+n+1)!}\R)^4\\ &G(m,n)=\frac{m+2n+2}{2}\frac{(-1)^mm!^6}{(2m)!}\L(\frac{n!}{(m+n+1)!}\R)^4 \end{align*}

ずなりたす。ペアが求たればあずは代入するだけです。
すなわち

\begin{align*} \sum_{m=0}^\infty F(m,0)=\sum_{n=0}^\infty \big(F(n,n)+G(n+1,n)\big) \end{align*}

より

\begin{align*} \zeta(3)=\frac{1}{2}\sum_{n=1}^\infty \frac{(-1)^{n-1}(205n^2-160n+32)}{n^5\binom{2n}{n}^5} \end{align*}

を埗たす。

投皿日2021幎12月30日

この蚘事を高評䟡した人

高評䟡したナヌザはいたせん

この蚘事に送られたバッゞ

バッゞはありたせん。

コメント

他の人のコメント

コメントはありたせん。
読み蟌み䞭...
読み蟌み䞭